4days : 2 weeks simplify​

Answers

Answer 1

Answer:

2:7

Step-by-step explanation:

ratio as a fraction in simplest form. 2 weeks = 14 days, so 4/14 = 2/7.

Answer 2

Answer:

2 : 7

Step-by-step explanation:

7 days = 1 week

y days = 2 weeks

[tex]\frac{1}{7} :\frac{2}{y}[/tex]

1 × y = 2 × 7

y = 14

4 days : 14 days

4 = 2 × 2

14 = 2 × 7

GCF = 2

4 ÷ 2 = 2

14 ÷ 2 = 7

2 : 7


Related Questions

pls help in this
7.6x5.2

Answers

Answer:

39.52

Step-by-step explanation:

Answer:

Table multiplication:

7.6 times

5.2 =

15.2

38.0

—-

39.52

" El ingreso mensual de una empresa está dado por la siguiente ecuación: (0,2) 10000(0,95) p I  , dondep es la cantidad gastada en publicidad:a) ¿Cuál es el ingreso total, cuando no se tienen gastos publicitarios?b) ¿Cuál es el valor del ingreso total, si $ 15 es el gasto mensual en promoción?"

Answers

Respuesta:

Ingresos = 9500

Ingresos = 8573,75

Explicación paso a paso:

La ecuacion:

Yo = 10000 (0,95) ^ (0,2) p

p = monto gastado en publicidad

Los ingresos totales cuando no hay gastos:

p = 0

Yo = 10000 (0,95) ^ (0,2) * 0

Yo = 10000 (0,95) ^ 0

Yo = 10000 (0,95)

Yo = 9500

Si el gasto mensual de la promoción es de $ 15

p = $ 15

Yo = 10000 (0,95) ^ (0,2) * 15

Yo = 10000 (0,95) ^ 3

Yo = 10000 (0,857375)

Yo = 8573,75

and angle is 21 more than twice its complement find it

Answers

Answer:

angle = 67°

Step-by-step explanation:

let

a = angle in degrees

(90-a) = complement

given

a = 2(90-a)+21

Solve for a

a = 180 - 2a + 21

3a = 201

a = 67°

someone help!!!!!!!!

Answers

Answer:

Top right

Step-by-step explanation:

First, we have "h is increasing on the interval (3, +∞)." This means that h must be increasing for all values 3 and higher. This is shown on the top left, top right, and bottom right graphs because when x goes up on each of them, the function increases as well. However, for the bottom left graph, this is not shown because as x goes from 3 to 4, the function decreases.

Next, the statement given is "h(4)=2". This means that when the x value is 4, the y value is equal to 2. This is true for the top left and top right pictures. However, for the bottom right, as the x value is 4 on the x axis, the corresponding y value is 1, not 2.

This leaves us with the top left and top right. The final characteristic given is "The domain of h is [3, +∞). This means that the x values of said function only go from 3 to infinity. As the top left graph has values of x below 3, this leaves the top right graph as h

plZ tell the answer fast
thank u​

Answers

Question:

Evaluate using a suitable identity:  (997)³

Answer:

991026973

Step-by-step explanation:

Given:

(997)³

To solve this using an appropriate identity;

i. Rewrite by splitting the value in bracket (997) into a difference of two numbers. i.e

(997)³ = (1000 - 3)³

ii. Use the appropriate identity

Since the split from (i) above gave a cube of the difference of two numbers,  we can apply the following identity for finding the cube of the difference of two numbers;

(a - b)³ = a³ - b³ - 3ab(a - b)

In this case;

a = 1000

b = 3

(iii) Substitute the values of a and b into the identity equation and solve;

(1000 - 3)³ = 1000³ - 3³ - 3(1000)(3)(1000 - 3)

(1000 - 3)³ = 1000000000 - 27 - 9000(997)

(1000 - 3)³ = 1000000000 - 27 - 8973000

(1000 - 3)³ = 991026973

Therefore,

(997)³ = 991026973

For each babysitting job, Tamar charges $6 for bus fare plus $8 per hour. She only accepts babysitting jobs if the total charge is at least $30. What is the minimum number of hours for a babysitting job she would accept?
2 and one-half hours
3 hours
4 hours
4 and one-half hours

Answers

Answer:

3 hours

Step-by-step explanation:

Let Tamar did the job for x hours,

Since, her charges for 1 hour = $ 8,

⇒  Her charge for x hours = 8x,

Also, the bus fare = $ 6,

hence, her total charge = Bus fare + Her charges for x hours

= 6 + 8x,

If the total charge is at least $30,

⇒ 6 + 8x ≥ 30

⇒ 8x ≥ 24

⇒ x ≥ 3

Thus, the minimum number of hours for a babysitting job she would accept = 3 hours

Hope this helps and if it does, don't be afraid to give my answer a "Thanks" and maybe a Brainliest if it's correct?  

Source: https://brainly.com/question/4172750

3 hours
Subtract 6 from 30 =24
24 divided by 8 = 3

HELP ME PLS- math question

Answers

Answer:

x+2x+90 =180

3x =180

x =60

x=2y

2y=60

y=30

2y +z =180

z =180-60

. z =120

For a function g(x), the difference quotient is StartFraction 6 Superscript x + h minus 3 Baseline minus 6 Superscript x + 3 Baseline Over h EndFraction. What is the average rate of change of g(x) on the interval from x = –2 to x = 1?

Answers

Answer:

The average rate of change in that interval is 1.99

Step-by-step explanation:

Here we have that the difference quotient for g(x) is:

[tex]\frac{6^{x + h} - 3 - 6^x + 3}{h}[/tex]

Remember that for a general function f(x), the difference quotient is:

[tex]\frac{f(x + h) - f(x)}{h}[/tex]

So if we look at the difference quotient for g(x), we can conclude that:

[tex]g(x) = 6^x - 3[/tex]

Also remember that the average rate of change in an interval (a, b) is just:

[tex]\frac{g(b) - g(a)}{b - a}[/tex]

So here we want the average rate of change of g(x) in the interval from x = -2 to x = 1, this is:

[tex]R = \frac{(6^1 - 3) - (6^{-2} - 3)}{1 - (-2)} = \frac{6 - 6^{-2}}{3} = 1.99[/tex]

The average rate of change in that interval is 1.99

Which number represents the probability of an event that is very likely to occur?
A. O 0.09
B. O 0.12
C. O 0.3
D. O 0.95

Answers

Answer:

D

Step-by-step explanation:

higher amount then the rest

Find f(-3) PLEASE HELP

Answers

Answer:

2

Step-by-step explanation:

When the point on the x-axis is on -3, the point on the y-axis is 2. Therefore, f(-3) is equal to 2.

If this helps please mark as brainliest

Answer: 2

Step by step explanation

angle2 cong angle5 so A|| B

Answers

Answer:

False

Step-by-step explanation:

If angle 2 and angle 5 are equal in length, then what we have is that the two lines are not parallel

For the two lines to be parallel, these two angles should be supplementary

what this mean is that these two angles should add up to give 180 degrees

In the diagram below of triangle PQS, S is the midpoint of PR and T is the midpoint of QR. If ST = 5x - 22, and PQ = 3x + 19, what is the measurement of PQ?

Answers

Answer:

PQ = 46

Step-by-step explanation:

The midsegment ST is half the length of the third side PQ , that is

ST = [tex]\frac{1}{2}[/tex] PQ , so

5x - 22 = [tex]\frac{1}{2}[/tex] (3x + 19) ← multiply both sides by 2 to clear the fraction

10x - 44 = 3x + 19 ( subtract 3x from both sides )

7x - 44 = 19 ( add 44 to both sides )

7x = 63 ( divide both sides by 7 )

x = 9

Then

PQ = 3x + 19 = 3(9) + 19 = 27 + 19 = 46

In the circle below, segment AB is a diameter. If the length of arc ACB is 6pi what is the length of the radius of the circle?

Answers

Answer:

12

Step-by-step explanation:

Length of the radius of the circle is equals to 6units.

What is length of the arc?

" Length of the arc is defined as the distance between two point as a part of the circumference of the given curve."

Formula used

Length of the arc = rθ

r = radius of the circle

θ = central angle

Radius = [tex]\frac{diameter}{2}[/tex]

According to the question,

Given,

Diameter = AB

Length of the arc = 6pi

Here central angle formed with diameter 'θ' = 180°

                                                                     = [tex]\pi[/tex]

Substitute the value in the length of the arc formula we get,

   [tex]6\pi = r \pi[/tex]

⇒ [tex]r = 6units[/tex]

Hence, length of the radius of the circle is equals to 6units.

Learn more about length of the arc here

https://brainly.com/question/16403495

#SPJ2

what is the common difference for this arthimitic sequence? -8, -13, -18, -23
a. 5
b. -5
c. -28
d. -21

Answers

b. -5 is the answerrrrrrrrrrrr

Which of the following is equivalent to (16^3/2)^1/2

6
8
12
64

Answers

The answer is 8 because 16^1.5 is 64 and 64^.5 is 8

can someone help me i dont understand​

Answers

Answer:

8900000 * 10^-3

Step-by-step explanation:

(8.9*10^-3)/(10^-6)

(8.9*10^-3)/(1/1000000)

8.9*10^-3* 1000000

8900000 * 10^-3

The system of inequalities below describes the relationship between Cori's age C and Livy's age L. C + L ≥ 43 C - 4L ≤ 3 Livy could be 13 years old and Cori could be 35 years old.

Answers

Answer:

Cori could be 35 years old.

Livy could be 8 years old

Step-by-step explanation:

Given

[tex]C + L \ge 43[/tex]

[tex]C - 4L \ge 3[/tex]

Required

Possible values of C and L

Make C the subject in: [tex]C - 4L \ge 3[/tex]

[tex]C \ge 3 + 4L[/tex]

Substitute for C in [tex]C + L \ge 43[/tex]

[tex]3 + 4L + L \ge 43[/tex]

[tex]3 + 5L \ge 43[/tex]

Collect like terms

[tex]5L \ge 43-3[/tex]

[tex]5L \ge 40[/tex]

Solve for L

[tex]L \ge 8[/tex]

Solve for C

[tex]C \ge 3 + 4L[/tex]

[tex]C \ge 3 + 4 * 8[/tex]

[tex]C \ge 35[/tex]

Which transformations could have occurred to map
AABC to AA"B"C?
O a rotation and a reflection
O a translation and a dilation
O a reflection and a dilation
O a dilation and a rotation

Answers

The transformations that could map ABC to A"B"C" are: (d) a dilation and a rotation

From the figure, we have the following highlights:

Triangle A"B"C is smaller than triangle ABCThe side lengths of both triangles are in equivalent ratioTriangle ABC is rotated about point C to form triangle A"B"C

The above highlights mean that:

The transformations that could map ABC to A"B"C" are dilation and rotation

Hence, option (d) is true

Read more about transformations at:

https://brainly.com/question/4289712

Someone help me with this please!

Answers

Answer: y=40

Step-by-step explanation: 70-30=40

Jodi has two bank accounts. Her parents started Account A for her. It currently has $100 in it, and Jodi deposits $20 into it each month. Jodi's grandparents started Account B for her. It currently has $300 in it, and her grandparents deposit $40 into it each month.

Answers

Answer:

(f+g)(x) = 60x + 400

Step-by-step explanation:

Given :

Amount in account A:

f(x) = 20x + 100

Amount in account B :

g(x) = 40x + 300

Total amount in Account A and B:

f(x) + g(x) = (20x + 100) + (40x + 300)

(f+g)(x) = (20x + 40x) + (100 + 300)

(f+g)(x) = 60x + 400

According to exponent rules, when we multiply two power with the same base we _______ the exponents. Example: c^6⋅ c^4

Answers

Answer:

Step-by-step explanation:

According to exponent rules, when we multiply two power with the same base we ADD the exponents.

[tex]c^{6}*c^{4} = c^{6+4} = c^{10}[/tex]

Brainiest will be given . Please help me

Answers

Answer:

If the shape of end face has n sides

Then, in total, there are n+2 faces

Using that formula

Faces in 20-sided polygon = n+2 = 20+2 = 22

Step-by-step explanation:

Pls vote as brainliest

you have five dollar bills and ten dollars bills for a total amount of $325. you have a total of 37 bills. A) write a system of linear equations that models the situation. B) how many of each type of bill do you have?

Answers

Answer:

nine $5 bills and twenty-eight $10 bills

Step-by-step explanation:

f = # of five dollar bills

t = # of ten dollar bills

system of equations:

t + f = 37

10t + 5f = 325

solve the first equation for 't' to get:

t = 37-f

substitute '37-f' to 't' in second equation to get:

10(37-f) + 5f = 325

370 - 10f + 5f = 325

-5f = -45

f = 9

t + 9 = 37

t = 28

The length of a rectangle is 18 cm and the width is 6 cm. A similar rectangle has a width of 2 cm.What is the length of the second rectangle? pls help im solving hw ._.

Answers

Given:

Two rectangles are similar.

Length of first rectangle = 18 cm

Width of first rectangle = 6 cm

Width of second rectangle = 2 cm

To find:

The length of the second rectangle.

Solution:

We know that the corresponding sides of similar triangles are proportional. So,

[tex]\dfrac{\text{Length of first rectangle}}{\text{Width of first rectangle}}=\dfrac{\text{Length of second rectangle}}{\text{Width of second rectangle}}[/tex]

Let x be the length of the second rectangle. Then,

[tex]\dfrac{18}{6}=\dfrac{x}{2}[/tex]

[tex]3=\dfrac{x}{2}[/tex]

[tex]3\times 2=x[/tex]

[tex]6=x[/tex]

Therefore, the length of the second rectangle is 6 cm.

8/-5. + -1/2 how do you do it because I really don't know

Answers

[tex]\implies {\blue {\boxed {\boxed {\purple {\sf { \frac{ - 21}{10} \:(or) \: - 2 \frac{1}{10} }}}}}}[/tex]

[tex]\large\mathfrak{{\pmb{\underline{\red{Step-by-step\:explanation}}{\red{:}}}}}[/tex]

[tex] \frac{8}{ - 5} + ( \frac{ - 1}{2} )[/tex]

[tex] \\= \frac{ - 8}{5} - \frac{1}{2} [/tex]

Since the denominators are unequal, we find the L.C.M (lowest common multiple) for both denominators.

The L. C. M for 5 and 2 is 10.

Now, multiply the L.C.M. with both numerator & denominator.

[tex]\\ = \frac{ - 8 \times 2}{5 \times 2} - \frac{1 \times 5}{2 \times 5} [/tex]

[tex] \\= \frac{ - 16}{10} - \frac{5}{10} [/tex]

Now that the denominators are equal, we can subtract the numerators.

[tex] \\= \frac{ - 16 - 5}{10} [/tex]

[tex] \\= \frac{ - 21}{10} [/tex]

[tex] \\= - 2 \frac{1}{10} [/tex]

[tex]\large\mathfrak{{\pmb{\underline{\orange{Mystique35 }}{\orange{♨}}}}}[/tex]

Answer:

-21/10

Step-by-step explanation:

-8/5. + -1/2

2(-8/5) + (-1/2)5

-16/10 - 5/10

-21/10

which of the following exponential functions represents the graph below?

Answers

Answer:

D

Step-by-step explanation:

We can start by taking a look at the x and y intercepts. When x=0, y=3. Plugging this into each function,

5 * 3^(0) = 5

3 * 5^(0) = 3

5 * (1/3)^(0) = 5

3 * (1/5) ^ (0) = 3

Therefore, our answer must be either B or D. Then, although y=0 isn't given on the graph, based on where the graph is heading, we can say that y is getting smaller as x gets larger. For a big number, such as 100, we know that

3 * 5^(100) is something big and 3 * (1/5) ^ (100) is something small*. Therefore, as y must get small, not big, as x gets larger, D is our answer.

* We know that this is something small because anything less than 1 to the power of something greater than 1 makes it smaller. For example, 0.5² is 0.25, which is smaller than 0.5

Help





Will





Give




BRIANLIST




Answer

Answers

v = s/t => average speed = 39,1/2,06= about 19 mi/hr, i guess

(Find m∠IGH) m∠IGH=

Answers

Answer:

angle IGH = 50 degree

Step-by-step explanation:

triangle GHI is an isosceles triangle because it's two sides are equal.

if angle I is 50 degree then angle G is also 50 degree becasue in isosceles triangle the base angles are equal.

2 to the negative 3 power times how much is 2 to the negative 1 power? Does the answer match your answer from question 3b? Why or why not?

Answers

Answer: 2^-3 = 1/8

2^-1 = 1/2

you have to multiply 2^-3 by 4 to get to 2^-1

Step-by-step explanation:

five fifth-grade teachers and six fourth-grade teachers ordered 2 large pizzas all together how much will each person need to pay.

Answers

Answer:

$5.50

Step-by-step explanation:

5+6= 11

11/2

Other Questions
Can you turn this into an indirect sentence, please? Tim, try a little harder, said the professor. Write a story that ends with the words:that experrence was one of the best things that ever happened to me. help plzzzzzzzzzzzz ? work out 8^2/3^2/3 means to the power of 2/3 g A company's flexible budget for 15,000 units of production showed sales, $60,000; variable costs, $22,500; and fixed costs, $17,000. The sales expected if the company produces and sells 19,000 units is (Do not round intermediate calculations): Right now I'm working on a project where I have to make my own civilization but I'm stuck and I don't know what to put in my 5 categories, so does anyone have any suggestion? HELP HELP HELP!!! Why would an investor choose to invest in speculative stocks when he or she could invest in blue-chip stocks instead? Listing The first column below identifies the different types of people wholived in ancient Rome. Write two facts about each person in the secondcolumn.People in Ancient RomeFactsWealthy Romans1.Poor Romans2.Fathers3. 4.WomenEnslaved Persons5. A cyclist travels at a rate of 12 kilometers per hour. What is the rate in kilometers per minute? How many kilometers will the cyclist travel in 20 minutes? Do not round your answer. wich best describes how mRNA is used by a cell (x-2)(-5x^2+x)=(x)(-5x^2)+(x)(x)+(-2)(-5x^2)+(-2)(x) is an exsample of Please help Ill give brainliest How tall is the table?120cm90cmI HELP ASAPWhich of the following is NOT a statistical question?a. How old are the members of the football team?b. Who is the captain of the football team?c. What is the average 40-meter dash time for players on the football team?d. How tall are the players on the football team? 4. Describe the physical environment ofmedieval cities, Can someone help me? It's urgent and thank you! Cos (2a) [tg (2a) - tg (a)] = tg (a) Es de identidades trigonomtricas y realmente no entiendo me ayudan? PLEASE ANSWER ASAP Mia randomally surveys students in her middle school to learn about their favorite type of novel. Of 40 respondents, 16 liked to read mystery novels. Based on Mia's data, how many of the 350 students in her middle school like to read mystery novels? PLEASE HELP ASAP!Type your response in the box.Jorge is writing a research paper about the positive and negative effects of cell phones in school. Below is a description of a source he has found for his paper.The author of the source was a teacher who now works for the states education department. She wrote the article last year and uses quotations and information from other reliable experts to support her ideas. She is writing about the negative effects of cell phones in schools.Evaluate the information about the source Jorge has found. Determine whether the source would be reliable for his paper. Explain your response.. HELPPPPP!!!!!!The kinetic energy of a moving objectequals half of its mass times the square ofits velocity. Write this as an equation, andsolve for the kinetic energy of a baseballwith a mass of 0.14 kilograms traveling 40meters per second. The kinetic energy isthe unlock number.Your answer